Convergent series with non-negative terms, a counter-example with negative terms

Click For Summary
The discussion centers on proving the convergence of the series formed by m_n = max{a_n, a_{n+1}} when the original series ∑ a_n converges with non-negative terms. A participant claims to have solved the first part using a theorem about the convergence of partial sums. However, they struggle to find a counter-example demonstrating that the convergence does not hold if the non-negativity condition is removed. They inquire about other ways a series can diverge beyond the partial sums growing unbounded. The conversation highlights the complexities of series convergence and the implications of altering term conditions.
Crossfader
Messages
2
Reaction score
0

Homework Statement



The terms of convergent series \sum_{n=1}^\inftya_n are non-negative. Let m_n = max{a_n, a_{n+1}}, n = 1,2,...

Prove that \sum_{n=1}^\inftym_n converges.

Show with a counter-example that the claim above doesn't necessarily hold if the assumption a_n\geq0 for all n\geq1 is dropped.


2. The attempt at a solution

I think I've solved the first claim using a theorem which claims if series converges then its partial sum converges as well. This holds assuming that I understood right the meaning of m_n=max{a_n, a_{n+1}}

I'm stuck with another one, frankly saying I couldn't find any counter-example.
 
Physics news on Phys.org
Crossfader said:

Homework Statement



The terms of convergent series \sum_{n=1}^\inftya_n are non-negative. Let m_n = max{a_n, a_{n+1}}, n = 1,2,...

Prove that \sum_{n=1}^\inftym_n converges.

Show with a counter-example that the claim above doesn't necessarily hold if the assumption a_n\geq0 for all n\geq1 is dropped.


2. The attempt at a solution

I think I've solved the first claim using a theorem which claims if series converges then its partial sum converges as well. This holds assuming that I understood right the meaning of m_n=max{a_n, a_{n+1}}

I'm stuck with another one, frankly saying I couldn't find any counter-example.
How can a series fail to converge? One way is if the partial sums get larger and larger without bound (or more and more negative). Is there another way that a series can diverge?
 
Question: A clock's minute hand has length 4 and its hour hand has length 3. What is the distance between the tips at the moment when it is increasing most rapidly?(Putnam Exam Question) Answer: Making assumption that both the hands moves at constant angular velocities, the answer is ## \sqrt{7} .## But don't you think this assumption is somewhat doubtful and wrong?

Similar threads

  • · Replies 2 ·
Replies
2
Views
2K
  • · Replies 3 ·
Replies
3
Views
1K
  • · Replies 2 ·
Replies
2
Views
2K
  • · Replies 1 ·
Replies
1
Views
2K
  • · Replies 1 ·
Replies
1
Views
1K
Replies
8
Views
3K
  • · Replies 4 ·
Replies
4
Views
1K
Replies
14
Views
2K
  • · Replies 2 ·
Replies
2
Views
2K
  • · Replies 4 ·
Replies
4
Views
2K